Nature d'une série

Bonjour
une idée pour étudier $\quad\displaystyle \sum \limits_{n\geq 2} \frac{1}{n^{1+\frac{1}{\sqrt{\ln n}}}}. $

Réponses

  • Comm' d'hab', développement limité en $ o(...)$.
    Bon solstice.
    Fr. Ch.
    21/12/2020
  • NON !! Mauvais conseil !
    Écrire ${n^{1+\frac{1}{\sqrt{\ln n}}}}$$=ne^{\sqrt {\ln n }}$ et comparer à une série de Bertrand.
    Pardon.
    Fr. Ch.
  • bonjour

    la série proposée est convergente, nous allons utiliser la transformation suggérée par notre ami Chaurien soit avec n > 2
    $$u_n = \frac{1}{n.exp\sqrt{ln(n)}}$$

    et comparer cette série à termes tous positifs et décroissants à la série de terme général avec n > 2
    $$v_n = \frac{1}{n.ln(n).\sqrt{ln(n)}}$$

    qui est convergente en effet : $\Sigma_2^n\frac{1}{p.lnp.\sqrt{lnp}}$ est équivalent à $\frac{2}{\sqrt{ln(n)}}$ pour n grand
    (comparaison avec une intégrale)

    or la fonction f définie pour x > 2 par $f(x) = exp(\sqrt{lnx}) - (lnx).\sqrt{lnx}$ est croissante
    et supérieure à $e^{\sqrt{2}}-2\sqrt{2} = 1,28...$

    en effet la dérivée $f'(x) = \frac{1}{2x\sqrt{lnx}}[e^{\sqrt{lnx}} - 3\sqrt{ln(x)}]$ est positive pour $x=e^3$ et au delà

    f(x) étant strictement positive entraîne l'inégalité pour n > 2 : $$0 < \frac{1}{n.exp\sqrt{ln(n)}} < \frac{1}{n(ln(n)\sqrt{ln(n)}}$$

    et donc la convergence de la série initiale

    cordialement
  • Je rappelle que : ${n^{1+\frac{1}{\sqrt{\ln n}}}}=ne^{\sqrt {\ln n }}$. On sait que pour $x$ assez grand on a : $e^{\sqrt x}>x^2$, d'où : $e^{\sqrt {\ln n }}>(\ln n )^2$.
    En conséquence : $u_n=\frac 1{{n^{1+\frac{1}{\sqrt{\ln n}}}}}$$=\frac1{ne^{\sqrt {\ln n }}}$$<\frac 1{n (\ln n)^2}$, série de Bertrand convergente, comme j'ai dit.
    Bonne journée.
    Fr. Ch.
  • Ou bien, plus pittoresque. La tangente au graphe de $y=\ln x$ au point d'abscisse $e$ a pour équation $y=\frac xe$. Il en résulte : $\ln x \le \frac xe$ pour tout $x>0$, égalité pour $x=e$. D'où : $e^x \ge x^e$, et par suite : $e^{\sqrt x} \ge x^{\frac e2}$, et $ne^{\sqrt {\ln n}} > n({\ln n})^{\frac e2}$. Comme $\frac e2>1$, on a encore une majoration par une série de Bertrand convergente.
    Bonne journée.
    Fr. Ch.
  • On peut aussi faire une comparaison avec l'intégrale $\displaystyle \int_1^\infty \frac{1}{x}e^{-\sqrt{\ln x}}\,dx=\int_0^\infty e^{-\sqrt{y}}\,dy$.
  • merci pour l'aide(tu)

    on montre quelle converge en utilisant le test de condensation de Cauchy
Connectez-vous ou Inscrivez-vous pour répondre.